Đến nội dung

Hoang Tung 126 nội dung

Có 1000 mục bởi Hoang Tung 126 (Tìm giới hạn từ 29-04-2020)



Sắp theo                Sắp xếp  

#577345 $\sum \frac{1}{(a+b)^{2}}\g...

Đã gửi bởi Hoang Tung 126 on 01-08-2015 - 07:50 trong Bất đẳng thức - Cực trị

Cho a,b,c dương. Chứng minh: 

$\sum \frac{1}{(a+b)^{2}}\geq \frac{3\sqrt{abc(a+b+c)}(a+b+c)^{2}}{4(ab+bc+ca)^{3}}$

p/s: trích "sữ dụng AM-GM để chứng minh bất đẳng thức"

Đặt $\frac{1}{a}=x,\frac{1}{b}=y,\frac{1}{c}=z$ .BDT

 

 $< = > \sum \frac{1}{(\frac{1}{x}+\frac{1}{y})^2}\geq \frac{3\sqrt{3\frac{1}{xyz}(\sum \frac{1}{x})}(\sum \frac{1}{x})^2}{4(\sum \frac{1}{xy})^3}< = > \sum \frac{(xy)^2}{(x+y)^2}\geq \frac{3\sqrt{\frac{3(\sum xy)}{(xyz)^2}}.\frac{(\sum xy)^2}{(xyz)^2}}{\frac{4(\sum x)^3}{(xyz)^3}}$
$< = > \sum \frac{(xy)^2}{(x+y)^2}\geq \frac{3\sqrt{3(\sum xy)}.(\sum xy)^2}{4(\sum x)^3}$  (1)

 

Theo Bunhiacopxki ta có : $\sum \frac{(xy)^2}{(x+y)^2}\geq \frac{(\sum xy)^2}{\sum (x+y)^2}=\frac{(\sum xy)^2}{2(\sum x^2+\sum xy)}$  (2)

 

 Do đó cần CM $\frac{(\sum xy)^2}{2(\sum x^2+\sum xy)}\geq \frac{3\sqrt{3(\sum xy)}.(\sum xy)^2}{4(\sum x)^3}< = > 2(\sum x)^3\geq 3\sqrt{3(\sum xy)}(\sum x^2+\sum xy)< = > 4(\sum x)^6\geq 27(\sum xy)(\sum x^2+\sum xy)^2$  (3)

 

   Đặt $\sum x^2=m,\sum xy=n= > m\geq n$ $,m,n> 0$.Do đó (3)

 

  $< = > 4(m+2n)^3\geq 27n(m+n)^2< = > 4(m^3+6m^2n+12mn^2+8n^2)\geq 27m^2n+27n^3+54mn^2$
$< = > 4m^3-3m^2n-6mn^2+5n^3\geq 0< = > 4m^2(m-n)+mn(m-n)-5n^2(m-n)\geq 0$
$< = > (m-n)^2(4m+5n)\geq 0$

(Luôn đúng )

 

 Do đó (3) đúng nên (1) đúng và ta có ĐPCM .

 

  Dấu = xảy ra khi $m=n< = > \sum x^2=\sum xy< = > x=y=z$




#577061 $\frac{a^3}{b^2-2b+3}+\frac{2b^3...

Đã gửi bởi Hoang Tung 126 on 31-07-2015 - 16:06 trong Bất đẳng thức - Cực trị

Cho $a,b,c$ là các số thực dương thoả mãn điều kiện $a^3+b^3+c^3=3$ 

  Chứng minh rằng :

             $\frac{a^3}{b^2-2b+3}+\frac{2b^3}{c^3+a^2-2a-3c+7}+\frac{3c^3}{a^4+b^4+a^2-2b^2-6a+11}\leq \frac{3}{2}$

Ta có : $\frac{a^3}{b^2-2b+3}=\frac{a^3}{(b-1)^2+2}\leq \frac{a^3}{2}$

    $\frac{2b^3}{c^3+a^2-2a-3c+7}=\frac{2b^3}{(c^3-3c+2)+(a^2-2a+1)+4}=\frac{2b^3}{(c-1)^2(c+1)+(a-1)^2+4}\leq \frac{2b^3}{4}=\frac{b^3}{2}$

  $\frac{3c^3}{a^4+b^4+a^2-2b^2-6a+11}=\frac{3c^3}{(a^4+a^2+1+1+1+1-6a^2)+(b^4-2b^2+1)+6}\leq \frac{3c^3}{6\sqrt[6]{a^6}-6a+(b^2-1)^2+6}\leq \frac{3c^3}{6}=\frac{c^3}{2}$

 

Cộng theo vế các BDT $= > P\leq \frac{a^3+b^3+c^3}{2}=\frac{3}{2}$

 

Do đó ta có ĐPCM. Dấu = xảy ra khi $a=b=c=1$




#576966 $(\sum sin\frac{A}{2})(\sum cot\...

Đã gửi bởi Hoang Tung 126 on 31-07-2015 - 08:05 trong Bất đẳng thức và cực trị

 Cho tam giác $ABC$ .CMR :

 

    $(sin\frac{A}{2}+sin\frac{B}{2}+sin\frac{C}{2})(cot\frac{A}{2}+cot\frac{B}{2}+cot\frac{C}{2})\geq \frac{9\sqrt{3}}{2}$

 

 

 

 

P/s: Lâu lâu chưa post




#576936 $P=\frac{x^{3}}{x+yz}+\frac...

Đã gửi bởi Hoang Tung 126 on 30-07-2015 - 22:21 trong Bất đẳng thức và cực trị

Với các số thực dương $x,y,z$ thỏa mãn điều kiện: $x+y+1=z$. Tìm giá trị nhỏ nhất của biểu thức: 

$$P=\frac{x^{3}}{x+yz}+\frac{y^{3}}{y+zx}+\frac{z^{3}}{z+xy}+\frac{14}{(z+1)\sqrt{(x+1)(y+1)}}$$

Theo Cauchy_Swatch ta có : $\frac{x^3}{x+yz}+\frac{y^3}{y+xz}\geq \frac{(x+y)^3}{2(x+y+yz+xz)}=\frac{(x+y)^3}{2(x+y)(1+z)}=\frac{(x+y)^2}{2(z+1)}=\frac{(z-1)^2}{2(z+1)}$  (Do $x+y=z-1$)

 

 Mà $\frac{z^3}{z+xy}=\frac{4z^3}{4z+4xy}\geq \frac{4z^3}{4z+(x+y)^2}=\frac{4z^3}{4z+(z-1)^2}=\frac{4z^3}{(z+1)^2}$

 

  $(z+1)\sqrt{(x+1)(y+1)}\leq (z+1).\frac{x+y+2}{2}=\frac{(z+1)(z-1+2)}{2}=\frac{(z+1)^2}{2}= > \frac{14}{(z+1)\sqrt{(x+1)(y+1)}}\geq \frac{28}{(z+1)^2}$

 

Từ đó $= > P\geq \frac{(z-1)^2}{2(z+1)}+\frac{4z^3}{(z+1)^2}+\frac{28}{(z+1)^2}=> 2P\geq \frac{(z-1)^2}{z+1}+\frac{8z^3}{(z+1)^2}+\frac{56}{(z+1)^2}=\frac{(z-1)^2(z+1)+8z^3+56}{(z+1)^2}=f(z)$

 

  Ta chứng minh $f(z)\geq \frac{53}{4}< = > \frac{(z-1)^2(z+1)+8z^3+56}{(z+1)^2}\geq \frac{53}{4}$
$< = > \frac{9z^3-z^2-z+57}{z^2+2z+1}\geq \frac{53}{4}< = > 36z^3-4z^2-4z+228\geq 53z^2+106z+53$
$< = > 36z^3-57z^2-110z+175\geq 0< = > 12z^2(3z-5)+z(3z-5)-35(3z-5)\geq 0$
$< = > (3z-5)(12z^2+z-35)\geq 0< = > (3z-5)(4z(3z-5)+7(3z-5))\geq 0$
$< = > (3z-5)^2(4z+7)\geq 0$ (Luôn đúng)

 

 Do đó $2P\geq f(z)\geq \frac{53}{4}= > P\geq \frac{53}{8}= > P_{Min}=\frac{53}{8}< = > \left\{\begin{matrix} x=y & & \\ z=\frac{5}{3} & & \\ x+y+1=z & & \end{matrix}\right.< = > \left\{\begin{matrix} x=y=\frac{1}{3} & \\ z=\frac{5}{3} & \end{matrix}\right.$




#576748 $\left\{\begin{matrix} a_1=1,a_2>1...

Đã gửi bởi Hoang Tung 126 on 30-07-2015 - 13:12 trong Dãy số - Giới hạn

Chứng minh rằng tồn tại đúng một dãy số nguyên $a_1,a_2,....$ thõa mãn điều kiện :

$\left\{\begin{matrix} a_1=1,a_2>1 \\ a_{n+1}^3+1=a_n.a_{n+2},n\in N \end{matrix}\right.$

 

Ta có : $a_{2}^3+1=a_{1}a_{3}=1.a_{3}=a_{3}$

 

  $a_{3}^3+1=a_{2}a_{4}= > a_{4}=\frac{a_{3}^3+1}{a_{2}}=\frac{(a_{2}+1)^9+1}{a_{2}}=a_{2}.A+\frac{2}{a_{2}}$

 

Để $a_{4}\in Z= > \frac{2}{a_{2}}\in Z= > 2\vdots a_{2}= > a_{2}=2$ ( Do $a_{2}> 1$)

 

-Từ dãy đề bài $= > a_{n}^3+1=a_{n-1}a_{n+1}$  ,với $n\geq 1,n\in N$

 

 Từ đó ta sẽ chứng minh dãy số sau gồm toàn các số nguyên và đó là dãy duy nhất thỏa mãn :

 

  $\left\{\begin{matrix} a_{1}=1,a_{2}=2 & \\ a_{n}^3+1=a_{n-1}a_{n+1} & \end{matrix}\right.$

 

 

- Với $n=1,2$ thay vào thấy thỏa mãn .

 

- Giả sử bài toán đúng đến $n=k= > a_{k+1}=\frac{a_{k}^3+1}{a_{k-1}}$    (1)

 

-Ta chứng minh bài toán đúng đến $n=k+1$

 

+ Thật vậy ,ta có cần chỉ ra số sau nguyên

 

    $a_{k+2}=\frac{a_{k+1}^3+1}{a_{k}}$ ,tức là $a_{2}\in Z$

 

Ta có : $a_{k+1}^3+1=(\frac{a_{k}^3+1}{a_{k-1}})^3+1=\frac{a_{k}^9+3a_{k}^6+3a_{k}^3+a_{k-1}^3+1}{a_{k-1}^3}$  (2)

 

Theo Công thức truy hồi thì $a_{k+1}=\frac{a_{k}^3+1}{a_{k-1}}= > a_{k}=\frac{a_{k-1}^3+1}{a_{k-2}}= > a_{k-1}^3=a_{k}a_{k-2}-1$  (3)

 

 Thay (3) vào (2) $= > a_{k+1}^3+1=\frac{a_{k}^9+3a_{k}^6+3a_{k}^3+a_{k-1}^3+1}{a_{k}a_{k-2}-1}= > (a_{k+1}^3+1)(a_{k}a_{k-2}-1)=a_{k}^9+3a_{k}^6+3a_{k}^3+a_{k}a_{k-2}$  (Do $a_{k-1}^3+1=a_{k}a_{k-2}$)   (4)

 

 Mà $(a_{k}^9+3a_{k}^6+3a_{k}^3+a_{k}a_{k-2})\vdots a_{k}$  (5)

 

 Nên từ (4),(5) $= > (a_{k+1}^3+1)(a_{k}a_{k-2}-1)\vdots a_{k}$ .Mà $(a_{k}a_{k-2}-1,a_{k})=1$

 

  $= > (a_{k+1}^3+1)\vdots a_{k}= > \frac{a_{k+1}^3+1}{a_{k}}\in Z= > a_{k+2}\in Z$

 

Do đó giả thiết qui nạp được chứng minh 

 

  Vậy dãy duy nhất thỏa mãn bài toán là $\left\{\begin{matrix} a_{1}=1,a_{2}=2 & \\ a_{n+1}^3+1=a_{n}a_{n+2} & \end{matrix}\right.,n\in N$

 

 

 




#574890 Chứng minh rằng:$\sum \frac{1}{\left [ a+b...

Đã gửi bởi Hoang Tung 126 on 23-07-2015 - 21:55 trong Bất đẳng thức và cực trị

Cho $a,b,c>0$ và $16(a+b+c)\geq \frac{1}{a}+\frac{1}{b}+\frac{1}{c}$.Chứng minh rằng:$\frac{1}{\left [ a+b+\sqrt{2(a+c)} \right ]^3}+\frac{1}{\left [ b+c+\sqrt{2(a+b)} \right ]^3}+\frac{1}{\left [ a+c+\sqrt{2(b+c)} \right ]^3}\leq \frac{8}{9}$

Theo BĐT Cosi cho 3 số ta có :

 

 $\sum \frac{1}{\left [ a+b+\sqrt{2(a+c)} \right ]^3}=\sum \frac{1}{\left [ (a+b)+\frac{\sqrt{a+c}}{\sqrt{2}}+\frac{\sqrt{a+c}}{\sqrt{2}} \right ]^3}$
$\leq \sum \frac{1}{27.(a+b).\frac{\sqrt{a+c}}{\sqrt{2}}.\frac{\sqrt{a+c}}{\sqrt{2}}}=\frac{2}{27}\sum \frac{1}{(a+b)(a+c)}=\frac{2}{27}.\frac{\sum (b+c)}{(a+b)(b+c)(c+a)}$
$=\frac{4(\sum a)}{27(a+b)(b+c)(c+a)}$  (1)

 

 Mặt khác ta lại có : $(a+b)(b+c)(c+a)\geq \frac{8(\sum a)(\sum ab)}{9}$ (2)

 

 Từ (1),(2) $= > \sum \frac{1}{\left [ a+b+\sqrt{2(a+b)} \right ]^3}\leq \frac{4(\sum a)}{27.\frac{8(\sum a)(\sum ab)}{9}}=\frac{1}{6(\sum ab)}$  (3)

 

 Ta có : $16(\sum a)\geq \sum \frac{1}{a}=\frac{\sum ab}{abc}= > \sum ab\leq 16abc(\sum a)\leq 16.\frac{(\sum ab)^2}{3}= > \sum ab\geq \frac{3}{16}$  (4)

 

 Từ (3),(4) $= >\sum \frac{1}{\left [ a+b+\sqrt{2(a+c)} \right ]^3}\leq \frac{1}{6.\frac{3}{16}}=\frac{8}{9}$

 

 Do đó ta có ĐPCM.  Dấu = xảy ra khi $< = > \left\{\begin{matrix} a+b=\frac{\sqrt{a+c}}{\sqrt{2}} & & & \\ b+c=\frac{\sqrt{a+b}}{\sqrt{2}} & & & \\ a+c=\frac{\sqrt{b+c}}{\sqrt{2}} & & & \\ a=b=c=\frac{1}{4} & & & \end{matrix}\right.< = > a=b=c=\frac{1}{4}$




#574177 $S_{a}+S_{b}+S_{c}+S_{d}\ge...

Đã gửi bởi Hoang Tung 126 on 20-07-2015 - 10:15 trong Hình học không gian

 Cho tứ diện $ABCD$ với $S_{a},S_{b},S_{c},S_{d}$ là diện tích 4 mặt của tứ diện và $r$ là bán kính nội tiếp tứ diện

 

    CMR : $S_{a}+S_{b}+S_{c}+S_{d}\geq 8r^2$




#572602 IMO 2015: Việt Nam xếp thứ 5 toàn đoàn với 2 Vàng, 3 Bạc, 1 Đồng

Đã gửi bởi Hoang Tung 126 on 15-07-2015 - 08:05 trong Tin tức - Vấn đề - Sự kiện

 Chúc mừng 




#572600 $f: \mathbb{R}\rightarrow \mathbb{R}...

Đã gửi bởi Hoang Tung 126 on 15-07-2015 - 07:55 trong Phương trình hàm

Xác định $f(x)$

$f: \mathbb{R} \rightarrow \mathbb{R}  : x^{3}f(x+3y)-f(x)= f(y) + f(3y)$ 

- Chọn $x=y=0= > 0-f(0)=2f(0)= > 3f(0)=0= > f(0)=0$

 

- Chọn $y=0= > x^3f(x)-f(x)=f(0)+f(0)=0= > x^3f(x)=f(x)= > f(x)=0$  (Do $f(0)=0$)

 

  Vậy $f(x)=0$ thỏa mãn bài toán




#572597 $\sum \frac{(c+a-b)^{4}}{a(a+b-c)...

Đã gửi bởi Hoang Tung 126 on 15-07-2015 - 07:51 trong Bất đẳng thức - Cực trị

Cho $a,b,c$ là ba cạnh của một tam giác . Chứng minh rằng

$\sum \frac{(c+a-b)^{4}}{a(a+b-c)} \geq \sum ab$

Đặt $\left\{\begin{matrix} b+c-a=x & & \\ a+c-b=y & & \\ a+b-c=z & & \end{matrix}\right.= > x,y,z> 0= > \left\{\begin{matrix} a=\frac{y+z}{2} & & \\ b=\frac{x+z}{2} & & \\ c=\frac{x+y}{2} & & \end{matrix}\right.$

 

Do đó $\sum \frac{(a+c-b)^4}{a(a+b-c)}\geq \sum ab< = > \sum \frac{y^4}{\frac{z(y+z)}{2}}\geq \sum \frac{(y+z)(x+z)}{4}< = > \sum \frac{y^4}{z(y+z)}\geq \frac{\sum z^2+3\sum xy}{8}$

 

Theo Bunhia và Cosi ta có :

 

 $\sum \frac{y^4}{z(y+z)}\geq \frac{(\sum y^2)^2}{\sum yz+\sum z^2}\geq \frac{(\sum y^2)^2}{\sum z^2+\sum z^2}=\frac{\sum y^2}{2}=\frac{4\sum y^2}{8}\geq \frac{\sum y^2+3\sum yz}{8}$

 

   Do đó ta có  ĐPCM




#572596 Chứng minh rằng : $\sum \frac{x^{2}+yz}...

Đã gửi bởi Hoang Tung 126 on 15-07-2015 - 07:45 trong Bất đẳng thức - Cực trị

Cho các số thực dương $x,y,z$ thỏa mãn $\sum \sqrt{x} = 1$

Chứng minh rằng : $\sum \frac{x^{2}+yz}{\sqrt{2x^{2}(y+z)}} \geq 1$

  Áp dụng các bdt Bunhiacopxki ,Cauchy-Swacth 

 

Ta có : $\sum \frac{x^2+yz}{\sqrt{2x^2(y+z)}}=\sum \frac{x^2}{\sqrt{2x^2(y+z)}}+\sum \frac{yz}{\sqrt{2x^2(y+z)}}$
$=\sum \frac{x}{\sqrt{2(y+z)}}+\sum \frac{yz}{\sqrt{2x^2(y+z)}}$
$=\sum \frac{x^2}{\sqrt{2x}.\sqrt{xy+xz}}+\sum \frac{y^2z^2}{xyz\sqrt{2(y+z)}}$
$\geq \frac{(\sum x)^2}{\sum \sqrt{2x}.\sqrt{xy+xz}}+\frac{(\sum yz)^2}{xyz\sum \sqrt{2(y+z)}}$
$\geq \frac{(\sum x)^2}{\sqrt{(2\sum x)(2\sum xy)}}+\frac{3xyz(\sum x)}{xyz\sum \sqrt{2(y+z)}}$
$\geq \frac{(\sum x)^2}{2\sqrt{(\sum x).\frac{(\sum x)^2}{3}}}+\frac{3(\sum x)}{\sqrt{12(\sum x)}}$
$=\sqrt{\frac{3(\sum x)}{4}}+\sqrt{\frac{3(\sum x)}{4}}\geq \frac{\sum \sqrt{x}}{2}+\frac{\sum \sqrt{x}}{2}$
$=\sum \sqrt{x}=1$
$= > \sum \frac{x^2+yz}{\sqrt{2x^2(y+z)}}\geq 1$

 

  Do đó ta có ĐPCM .Dấu = xảy ra khi $x=y=z=\frac{1}{9}$




#572488 Chứng minh: $\sum \frac{1}{1-a^2}+\su...

Đã gửi bởi Hoang Tung 126 on 14-07-2015 - 19:38 trong Bất đẳng thức - Cực trị

Cho $a,b,c$ là các số thực không âm thõa mãn $a^2+b^2+c^2=1$.

Chứng minh rằng

 

$\frac{1}{1-a^2}+\frac{1}{1-b^2}+\frac{1}{1-c^2}+\frac{1}{1-ab}+\frac{1}{1-bc}+\frac{1}{1-ca}\geq 9$

BDT $< = > \sum (\frac{1}{1-a^2}-1)+\sum (\frac{1}{1-ab}-1)\geq 3$

$< = > \sum \frac{a^2}{1-a^2}+\sum \frac{ab}{1-ab}\geq 3$
$< = > \sum \frac{a^2}{a^2+b^2+c^2-a^2}+\sum \frac{ab}{a^2+b^2+c^2-ab}\geq 3$

 $< = > \sum \frac{a^2}{b^2+c^2}+\sum \frac{ab}{a^2+b^2+c^2-ab}\geq 3$ (4)

 Theo BDT Bunhiacopxki ta có :

 

 $\sum \frac{a^2}{b^2+c^2}+\sum \frac{ab}{a^2+b^2+c^2-ab}$
$=\sum \frac{a^4}{a^2b^2+a^2c^2}+\sum \frac{(ab)^2}{ab(a^2+b^2+c^2-ab)}$
$\geq \frac{(\sum a^2+\sum ab)^2}{2\sum a^2b^2+\sum ab(a^2+b^2)+abc(\sum a)-\sum a^2b^2}$
$=\frac{(\sum a^2+\sum ab)^2}{\sum a^2b^2+\sum ab(a^2+b^2)+abc(\sum a)}$  (1)

 

 Ta cần chứng minh : $\frac{(\sum a^2+\sum ab)^2}{\sum a^2b^2+\sum ab(a^2+b^2)+abc(\sum a)}\geq 3$
$< = > (\sum a^2+\sum ab)^2\geq 3abc(\sum a)+3\sum a^2b^2+3\sum ab(a^2+b^2)$
$< = > (\sum a^2)^2+2(\sum a^2)(\sum ab)+(\sum ab)^2\geq 3abc(\sum a)+3\sum ab(a^2+b^2)+3\sum a^2b^2$
$< = > \sum a^4+abc(\sum a)\geq \sum ab(a^2+b^2)$
$< = > a^2(a-b)(a-c)+b^2(b-c)(b-a)+c^2(c-a)(c-b)\geq 0$

    BDT này đúng vì đó là Schur bậc 4 

 

Do đó $\frac{(\sum a^2+\sum ab)^2}{\sum a^2b^2+\sum ab(a^2+b^2)+abc(\sum a)}\geq 3$  (2)

 

  Từ (1),(2) $= > \sum \frac{a^2}{b^2+c^2}+\sum \frac{ab}{a^2+b^2+c^2-ab}\geq 3$  (3)

 

 Từ (3),(4) $= > \sum \frac{1}{1-a^2}+\sum \frac{1}{1-ab}\geq 9$

 

 Do đó ta có ĐPCM .Dấu = xảy ra khi $a=b=c=\frac{1}{\sqrt{3}}$




#571348 $\frac{l_{a}^2l_{b}^2l_{c}^2...

Đã gửi bởi Hoang Tung 126 on 11-07-2015 - 09:58 trong Hình học

    Bài toán : Cho tam giác nhọn $ABC$. Gọi $a,b,c$ là độ dài 3 cạnh của tam giác ; $l_{a},l_{b},l_{c}$ là độ dài 3 đường phân giác trong ; $r_{a},r_{b},r_{c}$ là bán kính đường tròn bàng tiếp các góc $A,B,C$ .Gọi $R$ là bán kính đường tròn ngoại tiếp $\Delta ABC$.

 

        CMR : $\frac{l_{a}^2l_{b}^2l_{c}^2}{a^2b^2c^2}\leq (\frac{r_{a}+r_{b}+r_{c}}{6R})^3$




#571169 $\sqrt[3]{a}+\sqrt[3]{b}+\sqrt[3]...

Đã gửi bởi Hoang Tung 126 on 10-07-2015 - 20:54 trong Bất đẳng thức và cực trị

   Bài toán : Cho các số thực dương $a,b,c$ thỏa mãn $a+b+c=3$ .CMR:

 

            $\sqrt[3]{a}+\sqrt[3]{b}+\sqrt[3]{c}+5\geq (a+b)(b+c)(c+a)$

 

 




#571080 Thảo luận về Đề thi và Lời giải của IMO 2015

Đã gửi bởi Hoang Tung 126 on 10-07-2015 - 16:26 trong Thi HSG Quốc gia và Quốc tế

Xin nêu lời giải bài số học(problem 2)

Đặt $ab-c=2^x(1);bc-a=2^y(2);ca-b=2^z(3)$(với x,y,z$\geqslant$0)

Do vai trò như nhau của x,y,z ta giả sử $x\geq y\geq z$. Do vai trò như nhau của a,b,c ta cũng được giả sử$a\geq b\geq c$(*)

Trừ (1) cho (2) ta có $2^x-2^y=ab-c-bc+a=(b-1)(a-c)$ chú ý rằng x lớn hơn bằng y và b$\geq 1$

do đó a lớn hơn c

Trừ (2) cho (3) tương tự suy ra b lớn hơn bằng a

Trừ (1) cho (3) ta lại có b lớn hơn bằng c.

Do đó $b\geq a\geq c$(**).

Từ (*) và (**) ta có a=b=c và x=y=z

Từ đó ta quy về giải Pt nghiệm nguyên dương sau:$a^2-a=2^x$(đơn giản với việc sử dụng biệt thức delta của tam thức bậc 2 là số chính phương)

từ đó suy ra a=b=c=1

Bạn đã giải sai ngay từ bước giả sử ,vì vai trò của 3 biến ko hề như nhau




#570910 Bánh canh chém gió về kì thi IMO 2015

Đã gửi bởi Hoang Tung 126 on 10-07-2015 - 08:16 trong Thi HSG Quốc gia và Quốc tế

Anh xem cái đề HSG thái bình là đủ hiểu

 Khó hơn mọi đề bình thường ,hj




#570906 Bánh canh chém gió về kì thi IMO 2015

Đã gửi bởi Hoang Tung 126 on 10-07-2015 - 08:03 trong Thi HSG Quốc gia và Quốc tế

Hôm nay bước vào ngày thi thứ nhất thì phải đúng ko mọi người 




#570905 Bánh canh chém gió về kì thi IMO 2015

Đã gửi bởi Hoang Tung 126 on 10-07-2015 - 08:02 trong Thi HSG Quốc gia và Quốc tế

Hình như anh Đăng cũng quê Thái Bình thì phải?

Năm nay thi IMO có 3 người quê gốc Thái Bình ,công nhận Thái Bình bá thật ,chiếm 1 nửa luôn




#570903 Bánh canh chém gió về kì thi IMO 2015

Đã gửi bởi Hoang Tung 126 on 10-07-2015 - 08:01 trong Thi HSG Quốc gia và Quốc tế

Thấy mặt anh Hoàn trên báo THTT số 454 kìa :luoi:

Ừ ,vì anh ấy đã tham dự IMo năm ngoái rùi mà nên đương nhiên là sẽ có




#570900 $2160+(x+y-z)^2$

Đã gửi bởi Hoang Tung 126 on 10-07-2015 - 07:29 trong Phương trình - Hệ phương trình - Bất phương trình

  Bài toán : Tìm tất cả các số nguyên dương $x,y,z$ biết chúng lập thành 1 cấp số cộng và thỏa mãn :

 

    $\frac{x^2(x+y)(x+z)}{(x-y)(x-z)}+\frac{y^2(y+z)(y+x)}{(y-z)(y-x)}+\frac{z^2(z+x)(z+y)}{(z-x)(z-y)}=2160+(x+y-z)^2$

 

 

 

 

 

 

p/s: Lâu chưa post số học ,post 1 bài cho vui 




#570842 $A=\frac{a+b+c+4abc}{1+4(ab+bc+ac)}$

Đã gửi bởi Hoang Tung 126 on 09-07-2015 - 21:40 trong Bất đẳng thức - Cực trị

    Bài toán : Cho các số thực dương $a,b,c$ thỏa mãn $a^2+b^2+c^2=\frac{1-16abc}{4}$. Tìm GTNN của biểu thức :

 

                    $A=\frac{a+b+c+4abc}{1+4(ab+bc+ac)}$




#570822 $f(x^2+f(y))=\frac{f^2(x)}{2}+4y$

Đã gửi bởi Hoang Tung 126 on 09-07-2015 - 20:41 trong Phương trình hàm

   Bài toán : Tìm tất cả các hàm $f:R\rightarrow R$ thỏa mãn với mọi số thực $x,y$ :

 

                       $f(x^2+f(y))=\frac{f^2(x)}{2}+4y$




#570664 $x+6\sqrt{xy}-y=6$

Đã gửi bởi Hoang Tung 126 on 09-07-2015 - 07:35 trong Phương trình - hệ phương trình - bất phương trình

    Bài toán : Giải hệ phương trình :

 

                   $\left\{\begin{matrix} x+6\sqrt{xy}-y=6 & \\ x+\frac{6(x^3+y^3)}{x^2+xy+y^2}-\sqrt{2(x^2+y^2)}=3 & \end{matrix}\right.$

 

 

 

 

 

P/s: Lâu lâu chưa post hệ ,post 1 bài cho vui (dù nó đơn giản )




#570634 $A=\frac{\sum cosA+cosAcosBcosC}{1+\sum co...

Đã gửi bởi Hoang Tung 126 on 08-07-2015 - 22:29 trong Bất đẳng thức - Cực trị

    Bài toán : Cho tam giác nhọn $ABC$ có 3 góc nhọn . Tìm Giá Trị Nhỏ Nhất của biểu thức :

 

            $A=\frac{cosA+cosB+cosC+cosAcosBcosC}{1+cosAcosB+cosBcosC+cosCcosA}$

 

 

 




#570614 $\sum \frac{a+b}{1-ab}\leq 3(\su...

Đã gửi bởi Hoang Tung 126 on 08-07-2015 - 21:19 trong Bất đẳng thức - Cực trị

ĐHV chơi quả đăng lại thế này thật hiểm quá!

http://diendantoanho...1-ableq-3sum-a/

 Hiểm bình thường thôi ,thấy hay đăng lại ý mà